Finding Expected Value of a Double-Exponential Distribution

  • Thread starter Thread starter corny1355
  • Start date Start date
  • Tags Tags
    Integration
Click For Summary
The discussion revolves around calculating the expected value of a random variable X with a double-exponential distribution defined by the density function f_x(x) = (1/2)e^(-p|x|). The expected value is derived from the integral ∫ x * f(x) dx, which is initially approached by integrating from 0 to infinity. Participants note that since the distribution is symmetric about zero, the expected value can be shown to equal zero despite the positive range of integration. Suggestions include evaluating the integral by splitting it into two parts and considering the symmetry of the function. Ultimately, the expected value of X is confirmed to be 0.
corny1355
Messages
1
Reaction score
0

Homework Statement



The random variable X has a double-exponential distribution with parameter p>0 if its density is given by

f_x (x) = (1/2)e^(-p|x|) for all x.

Show that the expected value of X = 0.

Homework Equations



I know that the expected value of a random variable x is

∫ x * f(x) dx

The Attempt at a Solution



We are told that f_x (x) = (1/2)e^(-p|x|)

So I'm guessing you have to do the following integral going from 0 to infinity:

∫ x * (1/2)e^(-p|x|) dx

But I'm unsure about how to compute this integral.
 
Last edited:
Physics news on Phys.org
If your sample space is [0,\infty), how could the average value of X be 0?
Also, there wouldn't be a need for absolute values if x couldn't be negative.
I`m sure that the problem implicitly assumes that X can take all values in R.

You could evaluate the integral by splitting it in two pieces.
There's a faster way though. Maybe drawing the graph of f will help.
 
Question: A clock's minute hand has length 4 and its hour hand has length 3. What is the distance between the tips at the moment when it is increasing most rapidly?(Putnam Exam Question) Answer: Making assumption that both the hands moves at constant angular velocities, the answer is ## \sqrt{7} .## But don't you think this assumption is somewhat doubtful and wrong?

Similar threads

  • · Replies 4 ·
Replies
4
Views
1K
  • · Replies 19 ·
Replies
19
Views
4K
Replies
7
Views
1K
  • · Replies 4 ·
Replies
4
Views
2K
Replies
6
Views
1K
Replies
7
Views
2K
  • · Replies 13 ·
Replies
13
Views
2K
  • · Replies 8 ·
Replies
8
Views
2K
Replies
2
Views
2K
  • · Replies 15 ·
Replies
15
Views
2K